已编辑

已编辑

我想并排显示两个优化问题,中间有一个等价箭头。我已经尝试过(参见下面的代码)使用对齐环境,但我无法处理约束。我想将约束直接放在目标函数下方,但正如您所见,一个约束不符合我的要求。此外,我想将“左”问题向左对齐,将“右”问题向右对齐。也许你们中有人可以帮助我。提前致谢!我的代码:

\documentclass[12pt, oneside]{article}                              
\usepackage{amsmath}
\usepackage{amssymb}
\usepackage[english]{babel}
\makeatletter
  \newcommand\flausr{\@fleqntrue}
\makeatother
\begin{document}
\begingroup 
\flausr
\begin{align} 
&\max_{c^{1}_{1},c^{2}_{2}}  \mathbb{E}(u(c^{1}_{1},c^{2}_{2}) & \hspace{2.5cm} \iff & \max_{c^{1}_{1},c^{2}_{2}} \pi_{1} u(c^{1}_{1}) + (1-\pi_{1}) \rho u(c^{2}_{1} + c^{2}_{2})\\ 
\mathrm{s.t.} &\, c^{1}_{1}=(1-I) + pRI &  \mathrm{s.t.} c^{1}_{1}=pc^{2}_{2} \notag \\ 
& \,c^{2}_{2}=\dfrac{(1-I)}{p}+RI \notag
\end{align} 
\endgroup
\end{document}

通过编译代码,可以得到: 在此处输入图片描述

第一个答案后进行编辑:

首先,感谢您的回答!但我还有一个问题。也许我在原帖中描述错误:我希望“右”问题的目标函数与右对齐,但约束不对齐。是否可以获得一种描述,其中约束显示在目标函数下方,就像我之前帖子中的“左”问题一样?

提前致谢!

答案1

已编辑

这更加微妙:以下内容有帮助吗?

array基本上,我在这里通过嵌套具有适当对齐的环境来实现这一点。

我通过为每个优化问题定义命令来处理过多的复杂性。

输出

在此处输入图片描述

代码

\documentclass[12pt,oneside]{article}                              
\usepackage{amsmath,amssymb}
\usepackage[english]{babel}
\newcommand\E{\mathbb{E}}
\begin{document}
\newenvironment{myOptPb}
{% before
  \bgroup
  \def\arraystretch{1.3}
  \begin{array}[t]{l}
}
{% after
  \end{array}
  \egroup
}
\def\myFirstOptPb
{
  \begin{myOptPb}
    \max\limits_{c^{1}_{1},c^{2}_{2}}  \E\big[u(c^{1}_{1},c^{2}_{2})\big] \\
    \text{s.t. } c^{1}_{1}=(1-I) + pRI                                    \\
  \end{myOptPb}
}
\def\mySecondOptPb
{
  \begin{myOptPb}
    \max\limits_{c^{1}_{1},c^{2}_{2}} \pi_{1} u(c^{1}_{1}) + (1-\pi_{1}) \rho u(c^{2}_{1} + c^{2}_{2}) \\
    \text{s.t. \quad } c^{1}_{1}=pc^{2}_{2}                                                            \\[3mm]
    \text{with : \quad } c^{2}_{2}=\dfrac{(1-I)}{p}+RI                                                 \\
  \end{myOptPb}
}
\def\myThirdOptPb
{
  \begin{myOptPb}
    \text{if } f(x) = x^2          \\
    \text{then } f'(x) = 2 \cdot x \\
  \end{myOptPb}
}

\thispagestyle{empty}
blabla blabla blabla blabla blabla blabla blabla blabla blabla blabla blabla blabla blabla blabla blabla blabla 
\begin{align*}
  \def\arraystretch{3}
  \begin{array}{l@{\hspace{1cm}}c@{\hspace{1cm}}r}
    \myFirstOptPb & \Longleftrightarrow & \mySecondOptPb \\
    \myThirdOptPb & \Longleftrightarrow & \myThirdOptPb  \\
  \end{array}
\end{align*}
blabla blabla blabla blabla blabla blabla blabla blabla blabla blabla blabla blabla blabla blabla blabla blabla 
\end{document}

原来的


这是做此类事情的一个经典方法。

希望我没有误解你想要的布局

如果您还有其他问题,请直接提问!

输出

在此处输入图片描述

代码

\documentclass[12pt,oneside]{article}                              
\usepackage{amsmath,amssymb}
\usepackage[english]{babel}
\newcommand\E{\mathbb{E}} % so we don't have to type that too many times
\begin{document}
\thispagestyle{empty}     % avoid pesky page numbering
blabla blabla blabla blabla blabla blabla blabla blabla blabla blabla blabla blabla blabla blabla blabla blabla 
\begin{align*}            % we are in math mode, we use an array environment 
  \def\arraystretch{1.3}  % increase line spread by 30 percent : sage advice from Moriambar
  \begin{array}{l@{\hspace{1cm}}c@{\hspace{1cm}}r} % 3 columns : l[eft] c[entered] r[ight] aligned with 1cm between each
    \max\limits_{c^{1}_{1},c^{2}_{2}}  \E\big[u(c^{1}_{1},c^{2}_{2})\big] & \Longleftrightarrow & \max\limits_{c^{1}_{1},c^{2}_{2}} \pi_{1} u(c^{1}_{1}) + (1-\pi_{1}) \rho u(c^{2}_{1} + c^{2}_{2}) \\
    \text{s.t. } c^{1}_{1}=(1-I) + pRI                                    &                     & \text{s.t. \quad } c^{1}_{1}=pc^{2}_{2}                                                            \\ [3mm]
                                                                          &                     & \text{with : \quad } c^{2}_{2}=\dfrac{(1-I)}{p}+RI                                                 \\
  \end{array}
\end{align*}
blabla blabla blabla blabla blabla blabla blabla blabla blabla blabla blabla blabla blabla blabla blabla blabla 
\end{document}

干杯,

相关内容